Wednesday, December 25, 2013

Flaw in the Reasoning Question Problem Set Answer Key


All answer keys in this book indicate the source of the question by giving the month and year the LSAT
was originally administered, the Logical Reasoning section number, and the question number within that
section. Each LSAT has two Logical Reasoning sections, and so the Section 1 and Section 2 designators
will refer to the first or second Logical Reasoning section in the test, not the physical section number of the
booklet.
Question #1. Flaw. October 2000 LSAT, Section 2, #6. The correct answer choice is (C)
As with all Flaw in the Reasoning questions, you must closely examine the relationship between the
premises and the conclusion. In this argument, the editorial concludes that the advice of the economic
advisor is untrustworthy and “the premier should discard any hope of reducing taxes without a significant
decrease in government services.” What support is offered for this position? Is a discussion of taxation
issued presented? Is a discussion of the cost of government service provided? Is the position of the
economic advisor dissected? No. According to the editorial, the only reason for ignoring the economic
advisor’s advice is that the advisor was convicted in his youth of embezzlement. This fact has no bearing
on the argument made by the advisor, and focuses instead on attacking the person making the argument.
This is a classic Source or ad hominem argument, and you should immediately seek an answer choice that
reflects this fact.
Answer choice (A): A proposal is not rejected in the stimulus; rather, a goal is advocated by the advisor
and then the author questions whether that goal can be met by examining the background of the advisor.
There is no discussion of a “particular implementation” that is likely to fail.
Answer choice (B): This answer fails the Fact Test because there is no discussion of “what could happen
otherwise” and no discussion of people’s fears.
Answer choice (C): This is the correct answer. The answer is a perfect description of a Source argument.
Answer choice (D): This answer describes an evidence error in which a lack of evidence for a position is
considered to hurt the claim. In the argument, the author improperly used evidence about the advisor, and
this mistake is the error in the argument. Even though this introduced a flaw into the argument, from the
author’s perspective this was an attempt to use evidence against a position to hurt the position. The
editorial did not state or indicate that there was a lack of evidence when forming the conclusion. Put
simply, the editor thought he had a reason that undermined the claim; no argument was made that there
was a lack of evidence.
Answer choice (E): This answer describes Circular Reasoning. But, because the argument in the stimulus
gives reasons for its position (albeit weak ones), the argument is not circular.
Question #2. Flaw. December 2001 LSAT, Section 1, #10. The correct answer choice is (C)
As always, look closely at the structure of the argument—specifically the relationships between the
premises and conclusion. This breakdown presents the pieces in the order given in the argument:
Conclusion: Cotrell is, at best, able to write magazine articles of average quality.
Subconclusion/
Premise: The most compelling pieces of evidence for this are those few of the numerous articles
submitted by Cotrell that are superior.
Premise: Cotrell, who is incapable of writing an article that is better than average, must obviously
have plagiarized superior ones.
Examine the language in the conclusion (“Cotrell is, at best, able to write magazine articles of average
quality”) and the premise (“Cotrell, who is incapable of writing an article that is better than average”). The
two are identical in meaning, and thus we have an argument with circular reasoning. Do not be distracted
by the plagiarism argument in the middle of the text—that is a tool used to physically separate the
conclusion and premise, making it harder to recognize that the two are identical.
Answer choice (A): The argument does not ignore the potential counterevidence to the conclusion. The
potential counterevidence is the few articles submitted by Cotrell that are superior, and the author dismisses
them by claiming they are plagiarized. Although the reasoning used to dismiss the good articles is flawed,
it is an attempt to address the evidence, and thus the argument cannot be said to “simply ignore the
existence of potential counterevidence.”
Answer choice (B): This answer choice describes an Overgeneralization. The answer is wrong because
the argument generalizes by dismissing the atypical occurrences (the superior articles), as opposed to
generalizing from them.
Answer choice (C): This is the correct answer, and one of several different ways to describe Circular
Reasoning (note that in the first problem in this set Circular Reasoning was an incorrect answer). More
often than not, when you see Circular Reasoning it will be an incorrect answer choice, but you cannot be
complacent and simply assume it will be wrong every time you see it. This problem proves that it does
appear as the correct answer on occasion.
Answer choice (D): This answer describes an Appeal to Authority. The answer fails the Fact Test because
there is no reference to the judgment of experts.
Answer choice (E): This answer is similar to answer choice (B). The answer starts out reasonably well—
“it infers limits on ability.” The argument does attempt this (depending on your definition of “infer”). But,
does the argument make this inference based on a “few isolated lapses in performance?” No, the argument
dismisses the few superior performances. In this sense the answer is Half Right, Half Wrong. Therefore, it
is incorrect.
Question #3. Flaw. October 2002 LSAT, Section 1, #23. The correct answer choice is (A)
The structure of the argument is as follows:
Premise: Food producers irradiate food in order to prolong its shelf life.
Premise: Five animal studies were recently conducted to investigate whether this process alters
food in a way that could be dangerous to people who eat it. The studies concluded that
irradiated food is safe for humans to eat.
Premise: These studies were subsequently found by a panel of independent scientists to be
seriously flawed in their methodology.
Conclusion: Irradiated food is not safe for human consumption.
The author uses the fact that the studies were flawed to conclude that irradiated food is not safe for human
consumption. Is this a reasonable conclusion? No. The studies purported to prove that irradiated food is
safe. The fact that the studies used flawed methodology should have been used to prove that the studies
did not prove that irradiated food was safe. Instead, the activist takes the argument too far, believing that
because the studies did not prove that irradiated food is safe, therefore irradiated food is not safe. This is
the third errorin the Errors in the Use of Evidence section, where “Some evidence against a position is
taken to prove that position is false.” Answer choice (A) perfectly describes this mistake.
Answer choice (B): Use the Fact Test to easily eliminate this answer. Although past studies were shown to
have methodological flaws, this evidence is not used to prove that methodologically sound alternatives are
impossible to achieve.
Answer choice (C): It’s true, the argument does fail to consider the possibility that a non-flawed study
might provide only weak support for its conclusion. But—and this is the critical question—is that a flaw in
the reasoning of the activist? No, it is perfectly acceptable for the author to ignore an issue (non-flawed
studies) that does not relate to his argument. Remember, the correct answer choice must describe a flaw in
the reasoning of the argument, not just something that occurred in the argument.
Answer choice (D): As with answer choice (C), the author has failed to consider the statement in this
answer choice. But is this a flaw? No. The fact that animal testing is widely done and the results are
accepted as indicative of possible problems with humans falls under the “commonsense information”
discussed back in Chapter Two. Testing products on animals is a current fact of life, and the author made a
reasoning error by failing to consider the possibility that what is safe for animals might not always be safe
for human beings.
Another way of looking at this answer is that it effectively states that the author has failed to consider that
there is a False Analogy between animals and humans. He fails to consider it because the analogy between
animals and humans is not false.
Answer choice (E): Again, the activist does fail to establish this, but it is not necessary since the
independent scientists only commented on the methodology of the study, not the irradiated food itself.
Question #4. Flaw. October 2001 LSAT, Section 2, #18. The correct answer choice is (B)
This argument contains an error of composition, one where the status-enhancing activities of most
scientists are said to prove that the scientific community as a whole acts to enhance its status.
Answer choice (A): This answer choice describes a compositional error, but not the one that occurs in the
stimulus. The stimulus makes a judgment about the scientific community as a whole whereas this answer
states that a judgment is made about each and every scientist. The community as a whole is different than
each and every scientist, and thus this answer is incorrect.
If you are thinking about the difference between the community as a whole and each member within the
community, consider this statement: “Our community is against stealing.” While that may be true, there
may also be individual members of the community who are thieves and have no qualm about stealing.
Answer choice (B): This is the correct answer, and the answer describes the correct compositional error
made by the philosopher.
Answer choice (C): The answer is wrong—the author does not presume that the aim of personal career
enhancement never advances the pursuit of truth. Consider the second sentence: “Accordingly, the
professional activities of most scientists are directed toward personal career enhancement, and only
incidentally toward the pursuit of truth. The portion that states “only incidentally toward the pursuit of
truth” indicates that the author allows for the possibility that career-enhancement activities can result in
activities that pursue truth, even if only a little bit. But, since that contradicts the force of never, this answer
is incorrect.
This is the most frequently chosen wrong answer, as about 20% of test takers select this choice.
Answer choice (D): This answer choice describes the Uncertain Use of a Term. The term “self-interested”
is used only once in the argument (at the end of the first sentence), and the remainder of the argument is
consistent with the generally accepted meaning of “self-interested” and uses that meaning unambiguously.
Answer choice (E): The argument in the stimulus is about the relationships of parts and wholes, not about
cause and effect. This answer, which describes reasoning from an effect in order to infer its cause, is
therefore incorrect. An example of the reasoning described in this answer choice would be: “We know this
window was broken this afternoon, and only one pane of the window was broken. We also know the pane
was broken by a circular object. Therefore, a baseball was the cause of the broken window.

Question #5. Flaw. June 2003 LSAT, Section 1, #12. The correct answer choice is (D)
The argument opens with the classic “some people claim” construction. The legislators claim that the
public finds many movies to be offensive, but the author rejects that position and concludes the legislators
have misrepresented public opinion. The author uses the results of a survey as evidence. At first glance this
argument looks very strong to most people, but then they encounter the question stem and realize there
must be an error. This is a critical moment: when the question stem indicates an error is present but you did
not realize one exists, you must go back to the stimulus and look for the error. Do not proceed to the
answer choices thinking that the answers will clarify or reveal the error to you! The answer choices are
designed to subtly draw your attention toward side issues, and it is far preferable that you find the error
first and then find the answer that correctly describes the error.
In this argument, the error occurs with the people that were surveyed in the poll. The last sentence reveals
that the survey did not use an unbiased sample: “the respondents see far more current movies than does the
average movie goer.” As you might imagine, individuals who attend a large number of movies are by
definition interested in the movies that are being shown, and are more likely to be aware of the level of
violence and accepting of it. These tendencies make the sample unrepresentative of the general
population—an error correctly described in answer choice (D). To help spot this error, note that the last
sentence of the argument indicates that the surveyed individuals saw more movies than the average
moviegoer. In other words, the survey respondents were not average.
Answer choice (A): This answer describes a Source argument. The author uses survey data to attack the
legislator’s position and does not attack the credibility of the legislators.
Answer choice (B): The conclusion is based on the results of a survey about public opinion, not on
subjective judgments of moral offensiveness.
Answer choice (C): The argument is not about what causes antisocial behavior, so it is not a flaw that the
argument fails to consider that violent movies increase the prevalence of antisocial behavior.
Answer choice (D): This is the correct answer.
Answer choice (E): The argument gives no data to suspect that the responses were not based on a random
sampling of movies seen. The error is instead that the people surveyed represented a biased sample.
Question #6. Flaw-FL. October 2003 LSAT, Section 2, #20. The correct answer choice is (B)
The premises of the argument contain a Formal Logic setup:
HD = hot days in Hillview
SUL = smog reaches unsafe levels
WBE = wind blows in from the east
SUL
HD
WBE
As you learned in Chapter Eleven, the combination of two “some” statements does not yield any
inferences. Yet, the author draws a conclusion ( SUL WBE ) on the basis of the relationship
and you must identify the answer that explains why this conclusion is incorrect.
Answer choice (A): There is no proof in the argument that the condition of WBE sometimes accompanies
smog reaching unsafe levels—that is the mistake made by the author. The answer would be more attractive
if it read as follows:
“mistakes a condition (WBE) that sometimes accompanies hot days in Hillview for a condition that
sometimes accompanies unsafe levels of smog”
Answer choice (B): This is the correct answer. When two “some” statements are joined, no inference can
be drawn because the group common to both may be large enough that the two sub-elements do not
overlap. For example, let’s say there are 10 hot days in Hillview (HD), 1 day when the smog reaches
unsafe levels (SUL), and 1 day when the wind blows in the east. Is it necessary that the 1 day when the
smog reaches unsafe levels is the same day that the wind blows in from the east? No, but the argument
concludes that is the case, and that error is described in this answer choice. For reference purposes, here is
the answer choice with each abstract item identified in parentheses after the reference:
“fails to recognize that one set (HD) might have some members in common with each of two
others (SUL and WBE) even though those two other sets (SUL and WBE) have no members in
common with each other”
Answer choice (C): This answer choice describes the Uncertain Use of a Term, but the argument is
consistent in its use of “unsafe.” Therefore, this answer is incorrect.
Answer choice (D): Each premise is plausible regardless of the truth of the conclusion.
Answer choice (E): The argument does not feature causal reasoning. The conclusion clearly states that the
two events happen together, but there is no attempt to say that one caused the other. If you chose this
answer, try to identify the causal indicators in the argument—there are none.
Question #7. Flaw. December 2003 LSAT, Section 1, #26. The correct answer choice is (B)
This is the second problem in this set to feature an Evidence error. In this problem, the astronomer falls into
the second error from the Errors in the Use of Evidence section, where “Lack of evidence against a
position is taken to prove that position is true.” Answer choice (B) describes this mistake.
The astronomer’s argument is structured as follows:
Premise: I have asserted that our solar system does not contain enough meteoroids and other
cosmic debris to have caused the extensive cratering on the far side of the moon.
Premise: My opponents have repeatedly failed to demonstrate the falsity of this thesis. Their
evidence is simply inconclusive.
Conclusion: They [my opponents] should admit that my thesis is correct.
Answer choice (A): The argument in the stimulus does not include a Source attack. There is a difference
between stating that an opponent’s argument is wrong (which is legitimate) and attacking the character of
that opponent (a Source flaw). Always look to see if the author attacks the person or the position; a
legitimate argument can sometimes appear questionable if the author uses weighted language such as, “My
opponents are deluded in believing that my thesis is incorrect.” Although that phrasing sounds like a
personal attack, it is just a very strong way of stating that the author’s opponents are incorrect, and it is not
a Source attack.
Answer choice (B): This is the correct answer.
Answer choice (C): The astronomer’s thesis asserts that meteoroids and other cosmic debris are not the
cause of the cratering on the far side of the moon. By definition, therefore, the astronomer allows for
alternate explanations of the cratering.
Answer choice (D): There is no presumption in the argument similar to the one described in this answer.
Answer choice (E): This answer describes the Uncertain Use of a Term, but the argument does not use
“meteoroids” in an inconsistent way.
Question #8. Flaw-CE. December 2002 LSAT, Section 2, #20. The correct answer choice is (D)
The argument contains a causal conclusion that asserts that good health is primarily caused by informed
lifestyle choices (education):
Premise: Some people believe that good health is due to luck.
Premise: However, studies from many countries indicate a strong correlation between good
health and high educational levels.
Conclusion: Thus research supports the view that good health is largely the result of making
informed lifestyle choices.
The author errs in assuming that the correlation mentioned in the second premise supports a causal
conclusion.
Answer choice (A): A disproportionate number of people (about one in three) select this answer. Does the
argument presume that to make an informed lifestyle choice a person must be highly educated? The author
certainly believes that high educational levels lead to informed choices, but the answer suggests that the
author thinks that the highly educated are the only people able to make an informed choice. The wording is
too strong and this answer is incorrect.
Answer choice (B): The author specifically notes that good health is largely the result of making informed
lifestyle choices. There is no mention of poor health, nor need there be since the argument focuses on a
correlation between good health and education. Thus, overlooking the possibility mentioned in this answer
choice is not an error.
Answer choice (C): The author does not make the presumption that informed lifestyle choices are available
to everyone, just that making good choices generally results in good health.
Answer choice (D): This is the correct answer. Remember, the error of causality is one with many facets,
and one of those errors is assuming that no third element caused both the stated cause and the stated effect.
This answer choice indicates that a third element (such as money) could cause both the conditions
described in the argument. Remember, if you know an error of causality occurred in the stimulus, look for
the answer that uses the words cause or effect! This is the only answer to do so, and it is correct.
Answer choice (E): Unlike many causal conclusions, the conclusion in this argument is not ironclad. The
author specifically says that the effect is largely the result of the cause, and that statement implicitly allows
other causes to lead to the effect, even if one does not make an informed lifestyle choice.

No comments:

Post a Comment